Đến nội dung

quantv2006

quantv2006

Đăng ký: 07-06-2016
Offline Đăng nhập: 07-08-2019 - 20:43
*----

#671466 2x2014=y2+z2

Gửi bởi quantv2006 trong 13-02-2017 - 16:04

3) giải phương trình

    x2-x-100($\sqrt{1+8000x}+1$)=0

4) giải hệ phương trình

     x+$\frac{3y+x}{x^{2}+y^{2}}$=3

     y-$\frac{y-3x}{x^{2}+y^{2}}$ =0

 

Câu 3 nếu là 100 thì chịu, hehe.

 

Câu 4:

 

- Xét x= 0 $\Rightarrow y=1$

- Xét y = 0 $\Rightarrow$ vô nghiệm

 

- Xét $x,y\not\equiv 0$, nhân (1) với y, (2) với x rồi cộng 2 vế ta có: 2xy+3=3y. Tính x theo y, thay vào (2) để giải tiếp được y = -1, x=3.




#671425 2x2014=y2+z2

Gửi bởi quantv2006 trong 13-02-2017 - 08:52

3) giải phương trình

    x2-x-100($\sqrt{1+8000x}+1$)=0

 

Câu 3 là 100 hay 1000 vậy bạn?




#671305 2x2014=y2+z2

Gửi bởi quantv2006 trong 12-02-2017 - 18:56

Bài 7: Giả sử x>=y thì từ 2 phương trình đầu đã có y>=x. Vậy x= y. Tương tự có y=z. Vậy x=y=z. Thay vào 1 phương trình bất kỳ có x=y=z=1.




#671002 [HSG] Hình 9

Gửi bởi quantv2006 trong 10-02-2017 - 15:54

giúp em bài này nữa ạ

 

Cho đường tròn (O), dây AB cố định không đi qua O. Lấy 2 điểm C và D thuộc AB sao cho AC = CD = DB. Các bán kính đi qua C và D cắt cung nhỏ AB tại E và F.

a) Chứng minh: AE < EF.

b) Một điểm M di động trên đường tròn (O), điểm P thuộc AM, điểm Q thuộc BM sao cho AP = BQ. Chứng minh đường trung trực của PQ luôn đi qua 1 điểm cố định.

Gọi K là điểm chính giữa cung AB có chứa M. Ta có KA = KB. Chứng minh tam giác KAP = tam giác KBQ $\Rightarrow$ KP = KQ. Vậy trung trực của PQ đi qua điểm K cố định.




#670997 Tam giác ABC nội tiếp (O) có AD, BE, CF đường cao đồng quy tại trực tâm H. EF...

Gửi bởi quantv2006 trong 10-02-2017 - 15:27

Tam giác ABC nội tiếp (O) có AD, BE, CF đường cao đồng quy tại trực tâm H. EF cắt BC tại T.lấy L sao cho A là trung điểm LD. Vẽ đường tròn đường kính LD cắt LT tại S. SD cắt (O) tại X. CMR: X trung điểm SD.

Bài này là bài chọn đội tuyển của Nam Định năm 2015-2016 thì phải.




#669064 Cmr: PS//OK

Gửi bởi quantv2006 trong 20-01-2017 - 19:29

NE cắt (BDE) tại F. Dễ thấy BF là đường kính của (BDE) nên M, D, F thẳng hàng.

 

BO cắt (O) tại G. BG là đường kính của (O), BF là đường kính của (BDE) nên F, G, S thẳng hàng.

 

GA// NE (cùng vuông góc với AB). AC= MN. Từ đó chứng minh tam giác AGC = tam giác NFM. Vậy GA//=NF hay AGNF là hình bình hành.

 

P là trung điểm của AN nên P là trung điểm của GF. Vậy 4 điểm F, G, P, S thằng hàng.

 

OK // GP nên OK //PS.




#669006 Topic ôn thi hình học vào cấp 3 chuyên

Gửi bởi quantv2006 trong 20-01-2017 - 11:31

Bài toán 10 (TTT2 số 165). Cho tam giác $ABC$ nội tiếp $(O)$ với $AB<AC$. Tiếp tuyến tại $A$ của $(O)$ cắt $BC$ tại $T$. $AD$ là đường kính của $(O)$. $DB$ cắt $OT,AT$ tại $E,F$. $EO$ cắt $(AEF)$ tại $G$. Chứng minh rằng tâm nội tiếp tam giác $AGB$ nằm trên $(O)$.

 

(Bài này đã hết hạn trên TTT2 nhưng mình thấy đáp án trên báo hơi dài, hôm qua có một bạn giải ngắn gọn hơn) 

 

Bài 10:

 

2017_01_20_112634.png

Gọi I là giao điểm của GO và (O).

 

Tứ giác AFEG là tứ giác nội tiếp nên góc $\angle AGO=\angle AFE$

 

Lại có góc $\angle AFE=\angle AFD=\angle OAB=\angle OBA $ nên $\angle AGO=\angle ABO$ hay AGBO là tứ giác nội tiếp. Do đó $\angle BGO=\angle AGO=\angle ABO$ hay GO là phân giác góc AGB.

 

Xét tam giác GAB có đường tròn ngoại tiếp (GAB) và O là trung điểm của cung AB không chứa G. OI = OA = OB nên I là tâm đường tròn nội tiếp tam giác GAB.




#668998 Topic ôn thi hình học vào cấp 3 chuyên

Gửi bởi quantv2006 trong 20-01-2017 - 11:12

Bài 7, câu 3: M là trung điểm của BC nên $\frac{AB}{AC}=\frac{DC}{DB}$

 

Tam giác ABC và AEF đồng dạng nên $\frac{AB}{AC}=\frac{AE}{AF}$

 

Ta có $\frac{CE}{BF}=\frac{DC}{DB}=\frac{AB}{AC}=\frac{AE}{AF}=\frac{NE}{NF}$

 

Từ đó có  $\frac{NF}{BF}=\frac{NE}{CE}$

 

Vậy $\frac{NQ}{BQ}=\frac{NP}{BP}$ hay PQ // BC

 

Mình nghĩ cách này gọn hơn.




#668750 BÀI TOÁN TIẾP XÚC VỚI ĐƯỜNG TRÒN CỐ ĐỊNH

Gửi bởi quantv2006 trong 18-01-2017 - 11:29

Bài này điểm $M$ khá đặc biệt , ở đây mình có thể tổng quát bài toán như sau : , cho tam giác $ABC$ nội tiếp $(O)$ , $B,C$ cố định , $A$ thay đổi trên cung lớn $BC$ , phân giác $AD$ , $M$ là 1 điểm cố định trên trung trực $BC$ , đường thẳng qua $D$ vuông góc $BC$ cắt $AM$ tại $I$ thì $(I,ID)$ luốn tiếp xúc với 1 đường tròn cố định khi $A$ thay đổi

Bài mở rộng, Gọi giao của OM với (O) tại E, với BC là P, với đường tròn (J) ((J) là đường tròn tiếp xúc với (I;ID)) là F. Khi đó ta có: PF.ME = PE. R

 

Do PE, R, ME cố định nên PF cố định. Vậy (BCF) cố định.




#668746 Tìm a, b, c biết a+b+c+d-3=ab và a+b+c+d-3=cd

Gửi bởi quantv2006 trong 18-01-2017 - 09:24

Từ a+b+c+d-3=ab, ta có (a-1)(b-1)=(c-1)+(d-1)

 

Tương tự ta có (c-1)(d-1) = (a-1)+(b-1)

 

Nhận thấy nếu bất kỳ số nào trong (a, b, c, d) bằng 1 thì các số còn lại cũng bằng 1. a=b=c=d=1 là 1 nghiệm của 2 phương trình trên.

 

Với a, b, c, d >1, đặt m=a-1; n=b-1, p=c-1, q=d-1. Ta có

 

mn=p+q (1); pq = m+n (2)

 

(lưu ý m, n, p, q nguyên dương)

 

- Nếu (m,n,p,q) >1 thì (m-1)(n-1)>0, mn>=m+n; pq>= p+q. Vậy mn>=(m+n)=pq>=p+q. Dấu '=' xảy ra khi mn=m+n và pq=p+q hay (m-1)(n-1)=1 và (p-1)(q-1)=1. Từ đó có m=n=p=q=2, hay a=b=c=d=3 .

- Nếu trong (m, n, p, q) có 1 số =1, giả sử m=1. Khi đó n=p+q; n+1=pq -> (p-1)(q-1)=2. Vậy n=5, p=2, q=3 hoặc n=5, p=3, q=2. Vậy (a,b,c,d) = (2,6,3,4); (2,6,4,3). Tương tự ta có (a,b) = (2,6); (6,2); (3,4); (4,3).




#668729 Topic ôn thi hình học vào cấp 3 chuyên

Gửi bởi quantv2006 trong 17-01-2017 - 22:19

Bài toán 2 (Thi thử chuyên KHTN 2013, vòng 2, đợt 3). Cho tam giác $ABC$ nội tiếp đường tròn $(O)$. $P$ là một điểm nằm trong tam giác $ABC$. Trung trực $CA, AB$ lần lượt cắt $PA$ tại $E, F$. Đường thẳng qua $E$ song song $AC$ cắt tiếp tuyến tại $C$ của $(O)$ tại $M$. Đường thẳng qua $F$ song song $AB$ cắt tiếp tuyến tại $B$ của $(O)$ tại $N$.

 

1) Chứng minh rằng $MN$ tiếp xúc $(O)$.

 

2) Gọi $MN$ cắt dường tròn ngoại tiếp các tam giác $ACM, ABN$ lần lượt tại $Q,R$ khác $M, N$. Chứng minh rằng $BQ$ và $CR$ cắt nhau trên $(O)$.

 

 

 

2017_01_17_220647.jpg

Bài toán 2:

 

Câu 1. AP cắt (O) tại điểm thứ 2 là D.

 

Do NF // AB nên NF vuông góc với OF tại D. Vậy BFON là tứ giác nội tiếp.

 

Do NF // AB nên góc $\angle NFB = \angle FBA =\angle FAB = \angle NFD$. Vậy góc $\angle DFB = 2.\angle DAB = \angle DOB$. Hay tứ giác BFOD là tứ giác nội tiếp.

 

Vậy B, E, O, D, N cùng nằm trên một đường tròn, do đó góc $\angle ODN = 90^0$ hay ND là tiếp tuyến của (O) tại tiếp điểm D.

 

Tương tự có MD là tiếp tuyến (O) tại tiếp điểm D. Do đó M, N, D thẳng hàng hay MN tiếp xúc với (O) tại D.

 

Câu 2. Góc $\angle ARQ = \angle ARN = 180^0 - \angle ABN = \angle ACB$.

 

Tương tự có góc $\angle AQR = \angle ABC$.

 

Vậy tam giác AQR và tam giác ABC đồng dạng. Từ đó có tam giác AQB và ARC đồng dạng.

 

Do đó $\angle ABQ = \angle ACR$. Nếu gọi K là giao điểm của QB và RC thì tứ giác ABKC là tứ giác nội tiếp. Vậy K nằm trên (O).




#668711 BÀI TOÁN TIẾP XÚC VỚI ĐƯỜNG TRÒN CỐ ĐỊNH

Gửi bởi quantv2006 trong 17-01-2017 - 20:49

:lol:  :lol:  bạn làm thế nào mà tìm được siêu vậy

Bài này mò là chính! Nói thật quên cách làm rồi, giờ chưa mò lại. Nếu cần thì mình ngồi thử lại!




#668708 Chứng minh di chuyển trên 1 đườnng cố định

Gửi bởi quantv2006 trong 17-01-2017 - 20:35

2017_01_17_202659.jpg

Em thử chứng minh đơn giản xem sao.

- AO1 cắt (O1) tại D, AO2 cắt (O2) tại E. Dễ thấy D, Q, E thẳng hàng.

 

- AO cắt DE tại K, cắt (O) tại điểm thứ 2 là G. Dễ thấy D, B, G thẳng hàng, E, C, G thẳng hàng.

 

- Gọi M là trung điểm của BC, AM cắt (O) tại điểm thứ 2 là N.

 

- Do AP là đường đối trung của AM nên góc $\angle BAQ = \angle CAM$. Vậy góc $\angle GDE = \angle NBC$. Tương tự có góc $\angle GED = \angle NCB$. Do đó ta có tam giác NBC và GDE đồng dạng.

 

- Tam giác NBC và GDE đồng dạng, M là trung điểm của BC, góc $\angle KGD = \angle MNB$ nên K là trung điểm của DE.

 

- Tam giác ADE có O1O2 // DE nên AK đi qua trung điểm của O1O2. Vậy trung điểm của O1O2 nằm trên đường AO cố định.




#668700 Cho $\bigtriangleup ABC$ nội tiếp đường tròn tâm $O....

Gửi bởi quantv2006 trong 17-01-2017 - 20:05

Nếu từ điểm M mà hạ MH vuông góc với BC thì đường AH là đường trong bài P17 của thày Hùng ở tạp chí Pi số 1!




#668513 Chứng minh: CK //HQ; Chứng minh: L, T, V thẳng hàng; Tính số đo góc TKC?

Gửi bởi quantv2006 trong 16-01-2017 - 08:40

Bài 2, hạ CK' vuông góc với EF, sau đó chứng minh K' nằm trên HN dễ hơn.